由买买提看人间百态

boards

本页内容为未名空间相应帖子的节选和存档,一周内的贴子最多显示50字,超过一周显示500字 访问原贴
Quant版 - one question urgent
相关主题
请教一个stochastic integral的问题An integral question
有没有人知道这个方程的解问个简单的martingale的问题
问一个two dimensional Brownian Motion 的问题问两个GS面试题
Need help in SDE problemsexpectation of brownian motion
A question asked请教一道Ito积分
弱问两个问题 (stochastic calculus)电面 高盛ISG in IMD
一道随机题问一个随机积分的问题
random walk type problem问个pricing的题
相关话题的讨论汇总
话题: int话题: shouldn话题: ds话题: iaw话题: question
进入Quant版参与讨论
1 (共1页)
t******m
发帖数: 255
1
We know that int_0^tW(u)du =N(0,t^3/3). But I need to calculate E[e^(iaW(t))
*the above integral]. Can anybody answer this question? Many Thanks.
d*j
发帖数: 13780
2
a is a constant ?
把那个e^复数, 想想 normal 分布 cos 和 sin 的对称性

))

【在 t******m 的大作中提到】
: We know that int_0^tW(u)du =N(0,t^3/3). But I need to calculate E[e^(iaW(t))
: *the above integral]. Can anybody answer this question? Many Thanks.

t******m
发帖数: 255
3
a is a constant

【在 d*j 的大作中提到】
: a is a constant ?
: 把那个e^复数, 想想 normal 分布 cos 和 sin 的对称性
:
: ))

x******a
发帖数: 6336
4
It is exp(a^2t/2).
one can evaluate the expectation in many ways, in particular,
one can calculate directly by writing down the integral since the
distribution of W(t) is known.

E[e^(iaW(t))

【在 t******m 的大作中提到】
: We know that int_0^tW(u)du =N(0,t^3/3). But I need to calculate E[e^(iaW(t))
: *the above integral]. Can anybody answer this question? Many Thanks.

t******m
发帖数: 255
5
sorry. I am a little confused. distribution of W(t)is known but does the
integral N(0, t^3/3) has anything to do with W(t)? Thanks

【在 x******a 的大作中提到】
: It is exp(a^2t/2).
: one can evaluate the expectation in many ways, in particular,
: one can calculate directly by writing down the integral since the
: distribution of W(t) is known.
:
: E[e^(iaW(t))

p******5
发帖数: 138
6
If W(u) is the Brownian motion, \int^T_0 W(u) du = N(0, T^3/3) .
p******5
发帖数: 138
7
My solution: assume w(s) is a Brownian motion,
E(\int^T_0 e^{i a w(T)} w(s) ds )
= \int^T_0 E(e^{i a w(T)} w(s)) ds
= \int^T_0 E(e^{i a (w(T) - w(s) + w(s))} w(s)) ds
= \int^T_0 E(e^{i a (w(s))} w(s)) ds for w(T)-w(s) independent of w(s)
I think it is not hard to get E(e^{i a (w(s))} w(s)) by properties of
characteristic function. The density function of w(s) is N(0, s)
x******a
发帖数: 6336
8
sorry,did not see the product...
paul1985 is correct.

the

【在 t******m 的大作中提到】
: sorry. I am a little confused. distribution of W(t)is known but does the
: integral N(0, t^3/3) has anything to do with W(t)? Thanks

t******m
发帖数: 255
9
thx

【在 p******5 的大作中提到】
: My solution: assume w(s) is a Brownian motion,
: E(\int^T_0 e^{i a w(T)} w(s) ds )
: = \int^T_0 E(e^{i a w(T)} w(s)) ds
: = \int^T_0 E(e^{i a (w(T) - w(s) + w(s))} w(s)) ds
: = \int^T_0 E(e^{i a (w(s))} w(s)) ds for w(T)-w(s) independent of w(s)
: I think it is not hard to get E(e^{i a (w(s))} w(s)) by properties of
: characteristic function. The density function of w(s) is N(0, s)

i****k
发帖数: 39
10

Why E(e^{i a (w(T) - w(s)}) term is dropped? Cannot see the connection
between "w(T)-w(s) independent of w(s)" and dropping the term. Could you
please give more detailed explanation? Thanks a lot!

【在 p******5 的大作中提到】
: My solution: assume w(s) is a Brownian motion,
: E(\int^T_0 e^{i a w(T)} w(s) ds )
: = \int^T_0 E(e^{i a w(T)} w(s)) ds
: = \int^T_0 E(e^{i a (w(T) - w(s) + w(s))} w(s)) ds
: = \int^T_0 E(e^{i a (w(s))} w(s)) ds for w(T)-w(s) independent of w(s)
: I think it is not hard to get E(e^{i a (w(s))} w(s)) by properties of
: characteristic function. The density function of w(s) is N(0, s)

y********l
发帖数: 11
11
Agree with idrunk. Shouldn't the integrand be:
E[e^{i a (w(T) - w(s)}]*E[e^{i a w(s)} w(s)]
= e^{a^2(T-s)/2}*E[e^{i a w(s)} w(s)]?
Thanks~

【在 i****k 的大作中提到】
:
: Why E(e^{i a (w(T) - w(s)}) term is dropped? Cannot see the connection
: between "w(T)-w(s) independent of w(s)" and dropping the term. Could you
: please give more detailed explanation? Thanks a lot!

a*********0
发帖数: 613
12
弱问:why int_0^tW(u)du =N(0,t^3/3)...
c********d
发帖数: 173
13
Shouldn't E[e^(iaW(t))=exp(-a^2t/2), rather than exp(a^2t/2)?
1 (共1页)
进入Quant版参与讨论
相关主题
问个pricing的题A question asked
【Stochastic Integral】一个简单问题?弱问两个问题 (stochastic calculus)
【Stochastic Integral】 \int_0^T W_tdt一道随机题
distribution of argmin{ Bt, t in [0, 1] } ?random walk type problem
请教一个stochastic integral的问题An integral question
有没有人知道这个方程的解问个简单的martingale的问题
问一个two dimensional Brownian Motion 的问题问两个GS面试题
Need help in SDE problemsexpectation of brownian motion
相关话题的讨论汇总
话题: int话题: shouldn话题: ds话题: iaw话题: question